No measureable curve of the earth?

0  2017-10-30 by 1tepa1

One often hears flat earthers say that there is no curve. That if you calculate the curve and go test it out in real life, you can see things in the horizon that you should not be able to see if there was a curve. And that bodies of water dont curve either and that some laser measurements over large canals show this. And that one can see distances on sea that one should not be able to see if there was a curve. Also the balloons that have been sent up with cameras that dont have fish eye lenses show a horizon that does not curve.

What do you think?

289 comments

sounds dumb honestly!

Many people have done these tests themselves though. Why is it dumb?

Because doing fallacious tests proves nothing other than your own ignorance, perhaps?

Have you looked at the way they test? For example over x amount of distance there is an y amount of curve. If the curve is such that the other shore should be hundreds of meters under the curve, how come it can be seen by naked eye?

Oh, dear. You really are serious aren't you? Those laser experiments are flawed and do not show a flat earth unfortunately. Looming is a very simple explanation for what is happening. But you can do a cool experiment yourself tonight. Find yourself a nice body of water and head down there for sunset. Lie on the floor with your heard on the ground and watch the sunset. As soon as the sun disappears below the horizon, jump up to your feet and you will see it set again. Because the Earth has a curved surface, as backed up by:

Millennia of sound mathematics.

The fact other celestial bodies are also spheres.

The fact that we have time zones, seasons, day and night etc.

The fact that if you travel in the same direction you eventually return to your destination.

The numerous human and machine observations of a clearly round Earth.

As soon as the sun disappears below the horizon, jump up to your feet and you will see it set again. Because the Earth has a curved surface, as backed up by:

Exactly the same thing would happen on a flat earth as well. The sun moves beyond the vanishing point of your vision which is due to perspective and when you rise up you will see farther and the sun will come back into view.

You don't know what those words you're using mean. If the perspective argument was true then the sun would change size greatly over the course of a day. It doesn't. You see more from a standing position precisely because the Earth is curved. Eesh.

If the perspective argument was true then the sun would change size greatly over the course of a day. It doesn't.

It does change size. Have you ever looked at how small it seems when it comes into view compared to how big it is when it is right over your head? Or how it shrinks when it goes back out from view.

It does not change size in any way like it would if any of the flat-earth theories were true. But, the funny thing is that you've actually got it mixed up. The Sun LOOKS bigger when it is on the horizon because of an optical illusion called the Ponzo Effect.

It does not change size in any way like it would if any of the flat-earth theories were true.

How should it change size?

It doesn't. It only appears to thanks to an optical illusion. Over the course of a year it appears to change size because of the Earth not having a perfectly circular orbit.

Done.

You said that according to the flat earth it would need to change size more than it does now. What do you base this statement on?

Because for a flat Earth to work, the Sun would have to be travelling over it's surface. That means that it would get closer and further away from you at different times of the day. So it would change its apparent size relative to your position on the Earth.

What do you think happens at night, by the way?

That means that it would get closer and further away from you at different times of the day. So it would change its apparent size relative to your position on the Earth.

Indeed and it does do that.

What do you think happens at night, by the way?

The sun is so far away that its light no longer reaches you.

Well, the sun doesn't change size throughout the day.

And the sun's light doesn't reach you any more? So there are no stars billions of miles away? All physics is wrong?

And the sun's light doesn't reach you any more? So there are no stars billions of miles away? All physics is wrong?

Yeah, stars would not be that far away. Space physics wrong.

Cool. So what is your evidence that physics is wrong?

You seeing a bit further that you think you should be able to isn't evidence of that, by the way.

Cool. So what is your evidence that physics is wrong?

I havent said they are.

Of course you have! You're claiming that light acts in a way which goes against our current understanding. All that science we've conducted in space is fake, so there's actually now no evidence for all that physics we've been using.

You're claiming that light acts in a way which goes against our current understanding.

Its still photons.

All that science we've conducted in space is fake, so there's actually now no evidence for all that physics we've been using.

Physics wasnt invented in space.

So how do you account for light which clearly comes from a long way away? How do you account for countless observations of the universe?

You cant have it both ways and just pick and choose your science. It's all built off of itself. One thing means this, therefore that. You throw in something like a flat Earth and the non-existence of gravity and all of our observations are null and void. All that science collapses.

So how do you account for light which clearly comes from a long way away?

Just because its of a different frequency does not imply distance.

How do you account for countless observations of the universe?

Like lights in the sky?

You throw in something like a flat Earth and the non-existence of gravity and all of our observations are null and void. All that science collapses.

Only the space science.

It's not just about 'frequency'. There are so many properties and different wavelengths in light. These differences give us information about where that light has come from. It is not from 'lights in the sky's.

'only the space science' makes no sense either. We are in space.

I think there is some merit to what they are doing but, are coming to the wrong conclusions ultimately. The earth is hollow and there are beings within in that transcend and craft our own perception of reality. We are not lowly humans on a ball earth of infinite nothingness. We mean something and are here for a reason. I'll convert any atheist or flat earther to hollow and possibly growing earth within one night of some drinking supplemented with some good weed.

How do you know there are beings down there? Transcend reality how?

I've taken enough mushrooms to know something is going on outside of our normal senses. I've met a few some were nice some were wicked. The wicked ones scared me. Not to mention reading the accounts of Admiral Richard E. Byrd.

I have also met beings. I dont take then to be real. Neither do I take this dream as real.

Hahahaha oh dear.

I'm curious to know, what are your thoughts on the relation between psychedelics and the ability to see these cock suckers in action? Any substance there?

Certain 'drugs' open your mind and re-wired the brain. It's no different than being blind without correct glasses and seeing perfectly with the right prescription glasses. Give your friend your glasses and it may be blurry as all. That's there experience. They need to find the 'right prescription'. Some find it in alcohol, weed, or psychedelics. The first two will take longer but, can lead to the same conclusion ultimately. Hope this makes sense.

Damn, that's an extremely good analogy. Thanks for your insight and reply.

Hollow Earth has the same problem as globe earth. No proof of curvature.

No it doesn't. The faulty curvature, the known curvature, mixed in with hollow earth, and growing earth makes sense for discrepancies.

Flat Earth also has en entrance at the north pole. So there's your hollow earth.

Other than that, I would love to see what proofs you have for the hollow model.

It says we live INSIDE a spinning ball right?

haha lets fight fire with fire, flat earth with hollow earth. cool, cool

So the only way you can convince someone is if they're drunk and stoned?

Sounds about right.

I'll convert any atheist or flat earther to hollow and possibly growing earth within one night of some drinking supplemented with some good weed.

I'm in.

I think gravity bends light. And when things are far away.. light always bends. Gravity goes magical things. It explains how we see stars behind the sun.. weird stuff

What about what spoken of in the OP?

Do you know what a scientific theory is? I'm assuming you don't have a car, or ever use one. You definitely don't use planes either do you? Because, why would you trust your life to a mere 'theory'. If I were you, I'd go and boil some water and put your hand in the pan. I mean, it's only Thermodynamics 'Theory' so you won't get burnt, right?

Those things can be observed. Gravity can not.

Hahahahahahahahaha! What are you talking about!?!?!? So you can observe the Second Law of Thermodynamics but you can't absorb gravity? Despite the fact that all you do all friggin day is observe gravity? Brilliant!

You can observe objects falling or rising. If the density of any object is more than the substance that it is in, it will sink. If the density is less, it will rise.

What makes a helium balloon rise up?

"Gravity" can hold all of the oceans of the earth stuck to a spinning ball that is flying through space but a helum balloon can defy it... An insect can take off but oceans are stuck.

Ouch.....my brain. Small children get this! Why cant you?

The reason there is an 'up' or 'down' is because of gravity. What direction does a helium balloon go in space? It doesn't go up or down because those terms don't apply. Things only rise and fall in their mediums due to their density BECAUSE OF GRAVITY. Without gravity there would be nothing to pull the objects down in the first place.

You've hit on a good point though. And that is that gravity is a very weak force. So weak that you i can raise my eyebrows with the muscles in my head. So weak that I can jump completely off the ground. But not so weak that it doesn't bring me back down again. That's the part you're forgetting about. Gravity works because of a cumulative effect. The larger the mass, the stronger the force. You need a huge object to generate enough gravity to have a planet such as ours. The difference between the oceans remaining on the surface (although that's not strictly true because of evaporation and wave action) and a fly taking off is that the fly is applying a force in opposition to gravity. The oceans are not.

Ouch.....my brain. Small children get this! Why cant you?

I am no longer a child that blindly believes everything they are told.

The reason there is an 'up' or 'down' is because of gravity.

That has not been proven. There could be an up and down just because there is an up and down. Gravity has never been found or observed. Before gravity everything was explained by density and boyancy, there was no need to have any magical forces pulling objects to each other.

Really? Because believing something as patently false and repeatedly disproven as the Flat Earth portrays you as EXACTLY the kind of person who believes what they're told.

Dude. Up and down are relative terms. Up and down are relative to the behaviour of objects on the Earth. Objects moving up and down on the Earth do so as a result of their interaction with the gravitational force.

Things WEREN'T explained by bouyancy and density beforehand. There was an attempt made by using the 'Aether' but that's outdated and proven to be incorrect. Hey, let's go back even further shall we? Before we knew more about science, everything was explained by Gods. No rain? Pray to the rain God! Sun rises in the morning? Being pulled on a chariot by Apollo! But we don't believe those things any more because we learned that they're not accurate. Just like the Earth being flat....

Objects moving up and down on the Earth do so as a result of their interaction with the gravitational force.

This has never been proven.

Things WEREN'T explained by bouyancy and density beforehand.

They were.

Dude.Whatever. Believe what you want to believe. Every day you use things that are the result of the very scientific discoveries you are claiming are false. We are having this conversation right now thanks to things you claim don't exist and you somehow think that you have managed to trump hundreds of years of scientific advancement. Laters.

Dude.Whatever. Believe what you want to believe.

Its a fact that gravity has never been directly observed. No one has ever seen this force of gravity, its only implied to exist.

And you can't prove your own existence either can you? Gravity is a force which can be observed thanks to the way it affects objects which interact with it. You can't directly observe magnetism but I don't hear you saying magnets aren't real (although you could be one of the ICP if your understanding of science is as it seems).

And you can't prove your own existence either can you?

I am existence and its obvious that existence is, otherwise there would be no question of proving.

You can't directly observe magnetism but I don't hear you saying magnets aren't real (although you could be one of the ICP if your understanding of science is as it seems).

You can see the magnetic fields if you throw in copper. You cant see gravitational fields.

But how can you predict your existence to me? How do I know you're not a computer? Or someone just pretending to be you? Now, it may be logical to assume that I am talking to a real person, but if I decide that I won't accept any evidence which comes from you yourself, then it becomes very hard for you to prove that you exist.

That is the attitude you are adopting in regard to well established science. You are just choosing to completely disregard some of the most brilliant mathematicians and physicists who have enriched your life in so many ways. I don't understand why.

But, you do understand about how you see magnetic fields. You're right. We don't see the field with our naked eyes. We can 'see' it by seeing how it interacts with objects. That is also exactly how we can observe gravity. We observe the effects it has over objects. Just as iron or copper filings are pulled into formation by a magnetic field, we are pulled to the Earth thanks to gravity.

But how can you predict your existence to me? How do I know you're not a computer?

You dont know, this could be and I suggest actually is a dream.

Or someone just pretending to be you?

All you know of me are the words you get, you dont even know me so why would you know or even care if someone is pretending to be me?

You are just choosing to completely disregard some of the most brilliant mathematicians and physicists who have enriched your life in so many ways. I don't understand why.

I like demonstrateable science, not mere theory or mathematical models.

I let Nikola Tesla speak for me:

Today's scientists have substituted mathematics for experiments, and they wander off through equation after equation, and eventually build a structure which has no relation to reality.

That is also exactly how we can observe gravity. We observe the effects it has over objects. Just as iron or copper filings are pulled into formation by a magnetic field, we are pulled to the Earth thanks to gravity.

You are just assuming there is a force called gravity. You can say there is gravity but you cant demonstrate it, you can just claim that what already is happening is proving you, which is circular arguing.

Things have always fell down in this manner, if you propose there is gravity, you need to be able to show it to me, show it affecting things. With magnets you can actually show two objects are attracted and there is no question, also you can reveal the magnetic fields themselves.

How are you not understanding this? The very fact that things fall to the ground is your proof. The fact that we observe the effects of gravity all through the universe if your proof. There is an attractive force is the universe which is related to mass and also to time, and we call it gravity. The word and idea didn't spring from nowhere. Gravity is easily observable and there have been countless experiments conducted. Tesla wouldn't be arguing your points for you dude.

Tesla had a different idea of gravity, this is what he said:

"... Supposing that the bodies act upon the surrounding space causing curving of the same, it appears to my simple mind that the curved spaces must react on the bodies, and producing the opposite effects, straightening out the curves. Since action and reaction are coexistent, it follows that the supposed curvature of space is entirely impossible - But even if it existed it would not explain the motions of the bodies as observed. Only the existence of a field of force can account for the motions of the bodies as observed, and its assumption dispenses with space curvature. All literature on this subject is futile and destined to oblivion. So are all attempts to explain the workings of the universe without recognizing the existence of the ether and the indispensable function it plays in the phenomena."

Great. But that's wrong.

And gravity is a belief.

I'm not aware of a situation where 'science' has been wrong. Science is a method. The method is sound, unless you think there's a better way of studying the universe than observation and constant testing and experimenting?

Theories have been wrong.

unless you think there's a better way of studying the universe than observation and constant testing and experimenting?

If anything I would like more actual observation and experimentation.

No one has observed gravity, we cant isolate it as a force and observe it. Its totally invisible to us.

Theories have been wrong!?! So that's you explicitly arguing against Flat Earth Theory?

And what do you mean we 'can't isolate gravity as a force'? It's a property of the universe.

Theories have been wrong!?! So that's you explicitly arguing against Flat Earth Theory?

If I say males are humans next you are going to ask me if I am arguing that all humans are males.

There isnt any flat earth theory, just proof that there is no actual measureable curve on earth.

And what do you mean we 'can't isolate gravity as a force'? It's a property of the universe.

That it is completely invisible. They say all mass has gravity, yet we cant observe how this force interracts between objects. We can see that the objects move yes, but we cant see the fields of gravity that supposedly come out of these objects and attract each other towards eatch other.

No, but you can't use a fatuous argument and not expect to have it thrown back at you. There's no proof of flat Earth. You haven't provided anything. You say you can't observe the curvature of the Earth, but only because you choose to ignore or deny the evidence. You've no proof of anything else. No pictures of the edge. No explanation for the observable universe. Nothing.

There's no proof of flat Earth. You haven't provided anything.

There is no measureable curve of the earth.

Haha..... Except for the ones taken from space, which apparently doesn't exist as far you're concerned. Anyway, enjoy yourself. I imagine it could be quite fun believing such a fantasy.

Isn't it strange how objects on earth don't attract each other? Take a large heavy object (lots of gravity in that) and put some small items next to it, they don't fly to it and stick.

Even in space the moon stays in an orbit, it doesn't get closer and closer until it collides with the earth, which it should since they are both attracting each other.

The "keeps missing the ground" explanation of orbiting is amusing, gravitational force acts on something but it doesn't get any faster or closer.

I just realized that, when I was 16, I read a book from the hitchhiker's guide series (or one of Douglas Adam's books, at least), and distinctly recall that it was mentioned, in a metaphysically humorous way, that the secret to flying is to jump off of something and then miss the ground.

I think it's time to revisit that series, I was just talking about it at a halloween party the other night, coincidentally.

Even in space the moon stays in an orbit, it doesn't get closer and closer until it collides with the earth, which it should since they are both attracting each other.

Also, at what size does a large mass not have influence on a smaller one. Does the moon have smaller masses attracted to it? Or is it too small?

And then there's the 3-body problem.

I like your posts, I've made a sub for non political conspiracies I could do with people like you. I'll invite more and maybe we can have some great discussions.

https://www.reddit.com/r/NonPolConspiracies

So what is the implication of this? What are you suggesting? How do you explain this without some type of force pulling on the water?

The water is more dense than the air. Earth itself is stationary and not a ball floating in space that makes everything relative.

I understand density makes things separate but, in your view, why wouldn't land separate into layers with air and water in between them?

Land is more dense than water and air.

Ok, but WHY does it sink? What is the force causing it to sink? Density is a property, not a force.

Why is there gravity? What is the force that is causing mass to have gravity?

We don't know everything about gravity.

What we do know is that there is SOME force that makes things that are more dense sink compared to things that are less dense. We know this because, as you said, dirt sinks in water. This is something we can observe and it simply would not be possible unless some type of force was causing the movement to happen.

This is the force that we call gravity. You can call it whatever you'd like really. You could call it "Banana Force" but the fact remains that we'd still be talking about the same thing.

This is something we can observe and it simply would not be possible unless some type of force was causing the movement to happen.

When I let go of a helium balloon, what force is pulling it upwards?

It's not a force pulling it upwards. Gravity is still pulling it downwards.

The difference is that helium is so much less dense that it does not have as much of an effect on the balloon filled with helium as it does on the dirt, the water, or even the regular atmospheric air that we breathe.

It's not a force pulling it upwards. Gravity is still pulling it downwards.

I thought you said there cant be any movement without any force acting on it. How can there be an upwards movement without it needing any other explanation than it being less dense but the downright movement cant be explained by it being more dense?

Every action has an equal and opposite reaction. By filling the balloon in with helium you have created a new space of lower density. It has the property of being lighter than the surrounding air.

The surrounding air gets pulled 'underneath' the balloon because the air is heavier. Since the balloon is filled with an area of lower density it floats on top of all the air that is being pulled down around it.

Why would it not work the same way in the downward direction?

I don't understand your question. Can you rephrase it?

In your earlier message you described how an object can rise up without referencing gravity. Why cant we use the same mechanic for the opposite direction?

I think you're mistaken. I never said an object can rise up unassisted without gravity.

Gravity is acting on everything constantly.

Yet there is zero actual evidence of gravity. Every other kind of force we can somehow measure. And by measure I dont mean calculate, but actually in real life measure or see to exist, to have some form.

The evidence is that everything in the universe behaves in a certain way and it is uniform and consistent.

Can you see other forces? How do you see heat? Fire? Ok what about things that are hot that aren't on fire?

How do you measure heat? A thermometer? Ok but a thermometer isn't a direct representation of heat but merely moving parts affected by a force.

Put simply, the only way to measure any force is by its effect on the world around it.

Gravity is measured and observed by its effect on the universe, just the same way as any other force. The evidence is that we see it happening in all day every day.

Gravity is measured and observed by its effect on the universe, just the same way as any other force. The evidence is that we see it happening all day every day.

Why cant we then find these fields of gravity that all mass has?

I'm not sure what you're referring to.

Offer an alternative to gravity that explains the phenomena we see.

No one has, thats the problem. you can say gravity is fake but FE scientist has ever sat down and quantified it...

Gravity is what they blame it on. No idea of why the relation ship is that way.

I think gravity bends light.

There is only one circumstance where it can be measured. If you do the research you will find about multiple expeditions during eclipses to try and prove that (and Einsteins theory). They couldn't find the distortion they expected and were about to announce as much until an urgent telegram told them not to publish, the establishment had decided it was going with Einstein.

I think I'm reading your question on a device that has a gps chip in it, which can receive timing signals sent by satellites at the speed of light so accurate that my phone can calculate the distance to those satellites based on the delay of the timing signal and then using a bit of geometry my phone can calculate my 3 dimensional coordinates anywhere on the surface of this spherical planet so accurately that some group of nerds in California was able to turn it into a game about catching Pokemon. What I'm still trying to figure out is why, with all this technology, anyone would still seriously post about flat Earth theory unless they were trying to make other conspiracy theorists look like idiots by association. Is that your goal here?

There is no conspiracy theorist bag you can group everyone in to. There are just people.

You didnt even adress the points of the OP.

There are no real pictures of satelites in space. If we have these satelites, why are we constantly building more and more towers?

Satelites were invented by a scifi author. Even if there are satelites, it does not explain the lack of curvature.

What are you, hourly?

If satellites aren't real how does my television talk to outer space?

Oh sorry, did I forget to address your question? Maybe you forgot that you are OP. Here let me try...

Yeah that's weird that I can't see China from where I live. Almost as if there was a planet in the way blocking my view.

Hey OP, do you believe China exists? Seems like my proof is pretty flimsy of China doesn't actually exist.

Your eyes cant see into infinity because on earth the atmosphere limits how far you can see. When there is fog you can barely see few meters in front of you.

How do you know your television talks to outer space instead of using the towers on earth to transmit the signal?

Is the sun inside our atmosphere?

I dont know if there is an inside and outside of what is called our atmosphere.

So what do you know?

That is quite a large scale question. Want to be more specific?

Dude get out of the city and watch the stars for an hour or two with no light pollution, you can literally see satalites moving in the sky with your naked eye.

You can see some stuff moving that you are told to be satelites.

Get an amateur telescope then, you can at least see the iss.

It looks like a hologram. The astronauts never do 360 turns on their spacewalks on the ISS with their cameras, it looks like a movie set.

I'm not a flat earther or hollow earther, but I have definitely seen some very suspect 'interview from the iss' videos that just look like a dude hanging in a harness upside down with the video inverted.

Also on space walks you can see bubbles coming from the astronauts helmet... as if they were actually in water.

It just so happens that nasa has a replica of the ISS under a pool of water so the astronauts can "train" there.

I appreciate you candidness here and am not trying to convince you of something more than the idea that there is deception and fraud happening at large scales, unfathomably large.

Follow your suspicions with the ISS, you may find the ride entertaining, at the least.

why are we constantly building more and more towers?

A cell phone isnt a satellite phone... You can get hybrids but they are pricey. Most consumer cells phones run off of cell towers, the call clarity is better and more reliable. Satellite phones are for very remote areas or for devices that need reliable signal.

The company I work for uses cell and satellite modems for their shit.

There are no real pictures of satelites in space.

There definitely are satellites. This one is easy to debunk. Many people and businesses rely on satellites, I can tell you for a fact that the company I work for rents time on one and the data it provides is only possible from a very high up orbit and its moving very fast.

Even satellites with closer vantage points and higher resolutions are out of luck. NASA's Terra and Aqua satellites carry the MODIS (or Moderate Resolution Imaging Spectroradiometer) instrument and operate only about 440 miles above the Earth’s surface.

“With the MODIS satellite, which has a resolution of one kilometer, you’d probably barely be able to make out something like the ISS," says Herman. "If it did pass through the field of view, you might see a bright spot, but you wouldn’t see many details, so it would be really hard to identify.” And that’s only if the ISS and Terra or Aqua happened to be in the same area.

Space is big and satellites are small. Also since we all know they exist why would someone send up a satellite worth tens of millions just to get pictures of other satellites? It would be a waste of money to do so, just to prove a point that doesnt need proving. There is more than enough info that satellites exist.

Could there be proof that satellites don't necessarily float in space but rather, say, tied to a high-altitude balloon, or is that too laughable to be possible?

I'm not saying that I know one way or another, but, in your opinion, is this a possibility?

Could there be proof that satellites don't necessarily float in space but rather, say, tied to a high-altitude balloon, or is that too laughable to be possible?

Not possible. Those balloons would have to be moving insanely fast, like impossibly fast to be possible.

So no, I do not think satellites are balloons.

Thanks for the quick response.

I'm not asking if they are balloons, but rather if they could be attached to balloons. Your thoughts?

I'm not asking if they are balloons, but rather if they could be attached to balloons. Your thoughts?

No, I thought I was fairly clear... The speeds involved are too high for balloons or really anything in the atmosphere.

The balloons would also be at the mercy of the winds so they wouldnt have the exact same predictable paths as we expect with a satellite. For example the company I work for rents time on a satellite and they have windows to take measurements with, requiring multiple passes of the same satellite. You can literally see it pass over the site, measure move past the site and we wait until it makes another pass from the opposite side and take more measurements. Repeat until measuring is done.

There is no feasible way for a balloon to replicate what that satellite does. Zero, never, no way.

Thank you for taking the time to respond.

I ask this because I trust that what you are saying would make sense, therefore, I'm curious what the purpose of balloon-launched satellites would be, as there seems to be solid claims that they exist. Why would they do that if the paths weren't predictable and if satellites in space could do it better? Thoughts?

I'm curious what the purpose of balloon-launched satellites would be, as there seems to be solid claims that they exist.

From my understanding the balloon is just to cut out a bulk of the flight. Those satellites still launch themselves from the balloon using a traditional rocket to get into orbit. There is no way for a balloon to deliver a payload that remains in orbit, its not possible.

Why would they do that if the paths weren't predictable and if satellites in space could do it better? Thoughts?

The only balloons we use are for weather monitoring, not geological surveying or communications. They are weather balloons.

Im not sure where you were going with this.

Im not sure where you were going with this.

I'm just trying to discern where your expertise does and doesn't contribute to this, as I do appreciate the contribution. Ultimately, I'm just offering information to add to the conversation, as satellites in space may not be completely necessary to prove or disprove anything.

It turns out that there are communication devices that do replicate what satellites do, that are strung from High-altitude balloons. And it is feasible to operate them at less of a cost than satellite tech.

From my understanding the balloon is just to cut out a bulk of the flight. Those satellites still launch themselves from the balloon using a traditional rocket to get into orbit.

You're speaking as if you have knowledge that that is how it happens. There isn't evidence of that. How did you come to that understanding, if I may ask?

The satellites that are currently tethered to balloons are not rocketed into space, they just stay attached to the balloons. They also don't require great speeds to collect or transmit data, there just needs to be a network of them, as you probably understand from your experience.

It turns out that NWS and international weather agencies launches 2 balloons every day from over 900 locations worldwide. If they can do it, so can government agencies with deeper pockets.

as satellites in space may not be completely necessary to prove or disprove anything.

No but you have to admit satellites are pretty damning to the FE movement.

It turns out that there are communication devices that do replicate what satellites do, that are strung from High-altitude balloons. And it is feasible to operate them at less of a cost than satellite tech.

I disagree. There is no simple way to replicate satellites without involving actual satellites.

This is ignoring the hundreds of businesses that create satellite equipment and the hundreds of thousands of businesses that help to create the parts that go into these satellites.

Show me one credible bit evidence as to what youre saying. Your whole argument about the satellites basically boils down to

"Its possible, right?"

Well no its not possible, its not a thought game its actually happening and being utilized by billions of people every day.

You really think they could hide a fake infrastructure as large as that?

You're speaking as if you have knowledge that that is how it happens. There isn't evidence of that. How did you come to that understanding, if I may ask?

I read it.

If your qualifier for information is first hand then how can you even begin to justify your hot air balloon theory?

There is zero evidence that is the case, but because I havent personally been a part of or experienced a rocket launched by balloon it means it doesnt exist?

You see the problem with your logic here, right?

Prove to me its not real or impossible.

Also I hate this argument because you arent applying it uniformly.

How can you tell bacteria or viruses exist? Have you seen them? How do you know penecillin works? Have you actually run the tests yourself and verified it? How do you know computers arent magic? Have you ever actually seen an electron move through a CPU?

So you shutting down my argument because I havent witnessed it first hand while at the same time giving me theories with zero evidence is intellectually dishonest.

The satellites that are currently tethered to balloons are not rocketed into space, they just stay attached to the balloons. They also don't require great speeds to collect or transmit data, there just needs to be a network of them, as you probably understand from your experience.

Like I said, not possible to replicate satellite info using balloons. Im sorry but its not possible under any scenario.

It turns out that NWS and international weather agencies launches 2 balloons every day from over 900 locations worldwide. If they can do it, so can government agencies with deeper pockets.

And no one has noticed that these are fake satellites?

Also I would love to see where you pulled that number from...

I disagree. There is no simple way to replicate satellites without involving actual satellites. This is ignoring the hundreds of businesses that create satellite equipment and the hundreds of thousands of businesses that help to create the parts that go into these satellites. Show me one credible bit evidence as to what youre saying. Your whole argument about the satellites basically boils down to "Its possible, right?" Well no its not possible, its not a thought game its actually happening and being utilized by billions of people every day. You really think they could hide a fake infrastructure as large as that?

I read it. If your qualifier for information is first hand then how can you even begin to justify your hot air balloon theory? There is zero evidence that is the case, but because I havent personally been a part of or experienced a rocket launched by balloon it means it doesnt exist? You see the problem with your logic here, right? Prove to me its not real or impossible. Also I hate this argument because you arent applying it uniformly. How can you tell bacteria or viruses exist? Have you seen them? How do you know penecillin works? Have you actually run the tests yourself and verified it? How do you know computers arent magic? Have you ever actually seen an electron move through a CPU? So you shutting down my argument because I havent witnessed it first hand while at the same time giving me theories with zero evidence is intellectually dishonest.

Alright, buckle in buddy. You clearly are not aware with a lot of things that you seem to throw around your opinion about, willy-nilly.

You're not familiar with how data-collecting instruments can and are employed around the world on high-altitude balloons, and are offering me nothing but your opinion on it without looking into it first. I am saying that it is possible to collect and transmit data that satellites collect without the need for them to go to space. It already is happening, in spades, and no-one's trying to hide it.

There is zero evidence that is the case, but because I havent personally been a part of or experienced a rocket launched by balloon it means it doesnt exist? You see the problem with your logic here, right? Prove to me its not real or impossible.

It's not a conspiracy just because you aren't familiar with it, and not being in the know does not equal zero evidence. It doesn't require that I prove it to you.

You've only proven to all that is reading this that you have been unable to ascertain such knowledge on your own and will call it illogical because it doesn't conform to your current set of knowledge.

I apologize for my frank tone, but you had made some assumptions about my ability to offer you EASILY google-able information. You could have just put my comments straight into google, if you wanted to validate my claims,

I will give you a chance to read up on that yourself on that before we continue, and I do mean that respectfully.

I am saying that it is possible to collect and transmit data that satellites collect without the need for them to go to space.

Sure but its not possible to replicate the speed. I get you can put sensors on a balloon but good fucking luck getting 300-400 miles up moving at 17k mph.

That is simple not possible to replicate. The company I work for uses satellites for topographical stuff and there is no fucking way that its a balloon barely a few miles off the ground.

No way.

It doesn't require that I prove it to you.

Thats a long winded way of saying you have no supporting evidence.

You've only proven to all that is reading this that you have been unable to ascertain such knowledge on your own and will call it illogical because it doesn't conform to your current set of knowledge.

And you are taking the intellectual high ground because you can no longer defend your points.

You are more than welcome to present actual evidence but all you have is theories with nothing to back them up. Which is why you are now approaching this from the angle of "Well you just dont get it" No I get it very well, more than yourself which is why this frustrates you.

I will give you a chance to read up on that yourself on that before we continue, and I do mean that respectfully.

I hate to sound like a jerk but... I know more about this than you do.

Please do not argue like this in the future it weakens your already fragile points.

Good luck.

(that last link shows that they are not yet launching balloons into space, so that contradicts your "understanding").

Weather balloons =/= rocket launching platforms or satellites.

Try again.

I didn't make that claim. I asked you if high-altitude balloons can carry satellites, you said no. I showed you otherwise.

Could there be proof that satellites don't necessarily float in space but rather, say, tied to a high-altitude balloon, or is that too laughable to be possible?

Not possible. Those balloons would have to be moving insanely fast, like impossibly fast to be possible.

From my understanding the balloon is just to cut out a bulk of the flight. Those satellites still launch themselves from the balloon using a traditional rocket to get into orbit.

You claimed that satellites launch themselves from balloons, but I was showing that you are making assumptions about things that you don't know anything about and was passing it off as knowledge of such things.

I didn't make that claim. I asked you if high-altitude balloons can carry satellites, you said no. I showed you otherwise.

Well actually as another user pointed out it hasnt been done yet so....

I man can ride in a balloon therefore airplanes are fake, right?

You claimed that satellites launch themselves from balloons, but I was showing that you are making assumptions about things that you don't know anything about and was passing it off as knowledge of such things.

Youre being pedantic because you cant argue your position anymore so instead you keep shuffling goalposts so forgive me if I forget where you want the ball kicked.

I think my post speaks for itself, 5 out of 5 dentists agree that I've done nothing but argue my position. Hopefully you'll be able to keep up next time. This post is public and everyone here gets to decide who did what, when, and how. Thank you for participating.

I have no problem standing by my actions and words, and, as you feel the desire to call me on specific claims that I've made, I'll be more than happy to continue to respond to them.

Hi,

don't forget project echo

https://www.nasa.gov/centers/langley/about/project-echo.html

They used it to communicate over distances that wouldn't otherwise be possible, by sending data from a ground station and bouncing it off the balloon.

"A 26 meter antenna (later named Echo) was built in Goldstone, California several miles from the Pioneer antenna to transmit voice communication. The Echo antenna included a transmitter operating at S-band (2.39 GHz), which was very sophisticated for the time. Another antenna was built in Holmdel, New Jersey to receive transmissions. Once the balloon was launched into orbit, a prerecorded message from President Dwight Eisenhower was transmitted from California and heard with clarity in New Jersey."

Yup, that one's a funny one. Thanks for reminding me and adding it.

I'm personally open to the idea of satellites being real but if someone wanted to fake it these could work. There are also ultra reflective blimps which I assume can be controlled better than a sphere.

Thanks for sharing your thoughts, I can totally respect that.

And yes, there are facebook and google blimps and high altitude gliders among others, and, for me, I used to often ask why would they go through the trouble when...well...technology. Google owns the Cold War era NRO satellite system and shouldn't need to progress to balloons and blimps unless there was a (good) reason, R&D-wise, to say the least.

The satelloons were used by govt agencies aswell as their purported other uses. One claimed usage was to monitor weather.

I find it hard to believe this massive beautiful beast is just for weather

http://i.imgur.com/dnLJWkU.jpg

I hear you, I find it nearly impossible to believe that bouncing signals off of its round surface produced predictable results, especially back then. But that is my un-researched opinion.

Thanks again, I appreciate that you brought this up, regardless of your position on it.

Well, they used it to communicate long distance. How reliably that was is anyones guess.

Some also had transmitters attached.

Word. All I had known is that they claimed to have used the reflective surface to bounce signals, but I hadn't come across any with transmitters, so I'll look into it further, thanks.

If I hadn't read that article I would have come to the same opinion as you, bouncing something off a sphere seems tricky.

If you consider the signal spread though it could be that a sphere is a good way of making a narrow beam cover a larger area once it returns to the ground.

That seems reasonable, yet almost beyond my understanding (I've studied a little bit of wave theory and radio propagation theory in my past vocation, but only paid enough attention to get the job done). I would seem to imply that there was technology to do that, wouldn't it? I don't know, I'm genuinely asking.

I did read that it was a passive system, which doesn't seem to support having transmitters, by definition. I am not doubting that you have seen somewhere that it does, but the NASA page at least seems to contradict that, not that I haven't seen them contradict themselves before. To be sure, I do appreciate your insight and perspective, thank you.

Yeah I misread the beacon transmitters as comms ones. More info:

The Echo 1A spacecraft was a 30.48 m diameter balloon of mylar polyester film 0.0127 mm thick. The spacecraft was designed as a passive communications reflector for transcontinental and intercontinental telephone (voice), radio, and television signals. It had 107.9 MHz beacon transmitters for telemetry purposes. These transmitters were powered by five nickel-cadmium batteries that were charged by 70 solar cells mounted on the balloon.

http://space.skyrocket.de/doc_sdat/echo-1.htm

It's interesting that these satellites survived up there for years.

I see how that could've caused confusion, I wouldn't have thought twice about it, either, and would have read it erroneously also, I'm sure.

It's interesting that these satellites survived up there for years.

Indeed. Also, the ability to bounce radio-telephone, much less television signals off of it, intercontinentally, without a booster or repeater, is, well, also interesting. I can't get a cell signal in my bathroom, for buddha's sake (not true, but it makes for a poor attempt at comic relief, at least).

Next time I see something shiny and spherical i'll bounce a flashlight off it and see if it works how I think it might.

Word. I have a laser and those Chinese meditation balls, gimme a sec.

No surprises, it goes all over the place with the slightest angle shift. But, I'm not presuming that a very strong signal does the same thing. Well, yes I am, I just don't know the facts, to be sure.

Since a laser is coherent in one direction it might not reproduce how radio waves work, spreading out as they travel.

I can dig it, thanks.

This diagram seems to suggest it would spread the beam, good for satellite tv etc.

https://d2gne97vdumgn3.cloudfront.net/api/file/uv91IfcFStN6PsNi82fu

Hmmm maybe the sun is just a big reflective sphere too ;-)

That's good info, thanks. I've had enough Echo 1 for today, thanks for the shares. And thanks for your other contributions ITT, I was reading up on some of them.

Also, here's something I've stumbled upon, trying to look into passively bouncing signals, until I decided I was over it, for the night.

I'm not sure if there's anything good in it, but maybe put it in your back pocket for a rainy day to see if it offers up anything about anything, whether or not it speaks to the topic at hand.

http://www.dtic.mil/dtic/tr/fulltext/u2/675512.pdf

I hadn't come across any with transmitters,

I take that back, I misread the article I think.

Word.

I have another conspiracy theory, that it's an alien spacecraft and they just attached tubes to it to pretend they made it ;-)

The ol' "Add-a-tube and call it non-alien tech" trick, I see. Kinda like the internets and it's series of tubes. But, I digress :)

Or just a massive party bong.

I'm in. :)

The first satellites came into existence just as they mastered project echo technology - satelloons.

Science.. **

How does science explain the lack of the curve?

It doesn't. It describes the curve extremely accurately and provides ample evidence to backup the fact that the Earth has a curved surface and is generally spherical.

Why then this curve has been measured to not exist by various people? Cities that should be behind the curve can be seen over bodies of water.

Nope. No inconsistencies whatsoever. There are plenty of people out there doing awfully bad science. But we shouldn't be listening to them. The evidence is just overwhelming and has been for millennia and I just don't get how there are still flat-earthers in this day and age!

There are flat earthers because people look up what science says for the curve to be over a particular distance and then they go out in the world and look and can see objects beyond the distance that they should see based on the amount of curvature science tells us exists in that distance.

You are talking about people doing bad science but you are not even looked into it. Most people who badmouth these claims from flat earthers never look into them, they are so unable to even entertain the idea that they never look into the evidence presented.

I'm sorry, but I have looked into it. I've liked into it because I have studied Geography and I have studied Physics and I understand and have tested these simple concepts. It's bad science of the flat-earther's part because they have decided that only their own personal observations count. It doesn't matter if some incredibly intelligent and skilled pilot has flown to the edge of space and has personally seen the curvature of the Earth. It doesn't matter that brilliant engineers and computer programmers and physicists have developed satellites which have pushed human technology forward. You haven't done it or seen it for yourself and because of that, you are willing to accept all sorts of mumbo-jumbo.

Just think. If the Earth was flat then I should be able to have a strong telescope set up on Mt. Kilimanjaro and see the Statue of Liberty. If the Earth was flat then I would be able to see the Southern Cross from anywhere in the world.

But you say it is flat. Now, your 'evidence' is based on a misunderstanding of the nature of perspective and the effect of topography on sight distances. BUT let's say we disregard that. Let's say that it doesn't add up and you might be onto something. So far you haven't shown that the Earth is flat. You've just shown that a calculation is off by what amounts to a pretty small margin. So what's your evidence FOR the hypothesis?

Just think. If the Earth was flat then I should be able to have a strong telescope set up on Mt. Kilimanjaro and see the Statue of Liberty. If the Earth was flat then I would be able to see the Southern Cross from anywhere in the world.

Not true because the air is a medium through which you cant see into infinity. You can only see so far through it. When the air is more humid and there is fog in the air the human eye cant see very far. The same kind of thing happens over longer distances.

So far you haven't shown that the Earth is flat. You've just shown that a calculation is off by what amounts to a pretty small margin.

When there is zero measureable curve, it can be called flat.

So how far can I see? And if I go higher and higher, do I get to a point where I just don't see anything at all because I'm too far away?

So how far can I see?

I dont know.

And if I go higher and higher, do I get to a point where I just don't see anything at all because I'm too far away?

Its not possible to not see anything at all if you are awake. There is some footage from high altitudes and its blue there.

So you don't know, but all these people who have performed experiments and have proven things beyond reasonable doubt are wrong. That's what you're saying

And we just go high enough that everything is just blue. Even though that's not what we observe....

Not really adding up.

So you don't know, but all these people who have performed experiments and have proven things beyond reasonable doubt are wrong.

What experiments? There have been no scientific evidence that the earth curves. No actual experiment. The stick and shadows thing works on a flat plane also.

Well as long as you continue to deny the entire field of space travel, satellites, space-based experiments and observations of the universe then what's the point of trying to discuss evidence with you?

The point was to talk about the points that were brought up in the original post.

yall need science in ya lives

How does science explain the lack of curvature?

Science doesnt explain the lack of curvature it explains THE curvature and might even go to the extent of explaining how a lazer is able to appear to be going in a "straight" line...i dunno just sayin'

Why is there then no measureable curve?

Erm. There is. It's pretty easy for any of the astronauts on the ISS to show you a curve. The friggin Greeks figured out the curvature of the Earth in the 3rd Century BCE. 400 years ago, Isaac Newton figured out the Earth isn't a perfect sphere.

It's just a scientific fact, mate.

Then why are people now trying to measure this curve in reality and cant find it?

Mate, there is so much footage and evidence out there. You're just being willfully ignorant. Who told you nobody 'does a 360 with the camera'? There are multiple cameras on the ISS which film from a variety of angles.

But then you don't even believe in space travel, so I don't know why I'm trying to explain any of this to somebody who denies such plain facts.

Who told you nobody 'does a 360 with the camera'?

There is no footage of that and I have watched spacewalks and never seen it nor do I know of anyone else that has seen it.

Mate, there is so much footage and evidence out there.

If you believe its real. I dont believe anything that comes from nasa because they fake stuff.

1)Here you go: https://www.youtube.com/watch?v=W9u297hArbI

2) So, you're evidence for the faked spacewalks is that you've not seen any 360 degree footage.... but if you had, then it still wouldn't be proof because all the footage is fake anyway. Got it.

There was no 360 in that spacewalk, we never even saw the camera turn around and show us the space station and the space that is behind it.

So, you're evidence for the faked spacewalks is that you've not seen any 360 degree footage.... but if you had, then it still wouldn't be proof because all the footage is fake anyway. Got it.

There are clear instances of them faking stuff. Such as wires being shown, gravity suddendly dissapearing (how is that possible if you are in space?), bubbles coming from the astronauts as if they were under water. Suprisingly they do have a space station replicant under water that they use to "train".

1) You have to click the screen and drag it for yourself.

2) No. There. Aren't. No wires. No bubbles. That's all fabrication and you have no proof for it.

No. There. Aren't. No wires. No bubbles. That's all fabrication and you have no proof for it.

There are videos up on youtube, you can look it up.

Yeah, I've seen easily debunked videos claiming to show wires, but not one of them shows any wires.

Have you seen the one where the gravity stops and the astronaut falls while being in the so called space station?

I've seen bullshit videos claiming to show stuff like that, but it's all bogus.

How is it bogus? Its clearly a nasa astronaut that has been on other official footage also.

I had a good search and watched a handful of videos but saw nothing at all out if the ordinary.

I didn't find a video like the one you're describing, but that sounds like it may be a video from the KC-135 Vomit Comet which simulates weightlessness through parabolic flight patterns. Which, by the way, is also how they would fake it if they were going to. Which makes the wires, greenscreen and underwater tanks seem even less plausible.

Which, by the way, is also how they would fake it if they were going to. Which makes the wires, greenscreen and underwater tanks seem even less plausible.

Many people do think they use all of those techniques.

Yes. And I'm saying that nobody needs to use those techniques. And you're getting awfully convoluted now.

Anyway, it's hilarious hearing this stuff.

How do you explain a flat earth but every other plannet and moon in the universe is a sphere?

Oh, that's just part of the ruse. If all the planets and celestial bodies around us are round, it's easier to convince us that ours is round, too, right? I find the best argument to convince us it's round is the curvature of the earth.

Flat earthers dont accept that they are round.

What are they then?

Light in the sky is all we can observe of them.

Except it's not. People and robots have traveled to the moon and brought back samples. We've sent probes to comets as well as to every planet in the solar system. You can't do that to lights in the sky.

If one believes we have been to the moon.

Well I wouldn't be dumb enough to believe something as ludicrous as the faked moon landings.

If we went to the moon, why are there no marks below the moon module? Why was the soft dust that the astronauts feet could stirr and leave foorprints on not affected in any way by the module descending? Even where its landing feet are there are no marks.

We cant even get to the moon now with our technology, nasa says they "lost" the tehcnology to go to the moon. We have a bigger computer in our smartphone than the one they claimed to go to the moon with and now they claim it was some magical technology that they lost and would need to invent again in order to get to the moon...

None of what you have said is true. Just so you know. All of those weak arguments have been debunked THOROUGHLY over the last 40 years. No need for me to do it again.

Getting to the moon today would be easier than ever. Unfortunately, the political will and economic backing simply isn't there now.

Getting to the moon today would be easier than ever. Unfortunately, the political will and economic backing simply isn't there now.

If it is easier than ever, why does nasa say they have lost the technology to go there? You can look up videos right now from youtube from nasa emplyees that said this, it has not been debunked as you say. They literally said it, its not something I made up.

No need for me to do it again.

What I said has not been debunked. You are just making shit up because you dont really care about what is true, only about being right.

The ability to build machines to go to the moon still exists. But the old technology which was originally used has been destroyed/lost. The factories were re-purposed. The moulds for the parts were destroyed. The machines built today would use different materials and different components so we simply wouldn't be able to build a Saturn V today. Well, you could, but it would be prohibitively expensive and a bit of a waste of time.

In this case (with you denying historical and scientific fact), I have the luxury of being both on the side of truth and being right.

There is no way in which the moon module can ascend to the surface of the moon while not leaving any marks in the dust like surface of the moon.

It landed on solid rock which was covered with a thin layer of dust. There are marks, just not a big crater or anything.

How did the reflectors get up there?

And yes I know they exist because I helped write code for a college project involving them.

How did the reflectors get up there?

What proof do you have that they actually exist?

As part of a college project the software devs teamed up with the engineering students to build a laser that bounced a signal off the moon, recollect the data and interpret it in various ways.

One of which was how long of a delay after sending vs receiving. Since we know the speed of light we know how far away those reflectors were.

Which college?

Did you know that reflectors weren't supposedly needed in the first place (it just made it "better"), so it seems unnecessary to invoke reflectors placed on the moon as proof?

Did you know that reflectors weren't supposedly needed in the first place (it just made it "better"), so it seems unnecessary to invoke reflectors placed on the moon as proof?

You couldnt get the level of precision necessary to do what we did with shining light on ostensibly concrete...

I know the moon is reflective because you can see it every night reflecting light.

Edit: And, if you knew that, would you be kind enough to share that from your position of having worked on it, which I do trust that you have.

What?

I'm just stating that your seem to infer that reflectors on the moon prove that we've been to the moon. I'm stating that there doesn't need to be reflectors on the moon to shoot a laser at the moon, and most people don't know that.

I know the moon is reflective because you can see it every night reflecting light.

I agree with you somewhat. To clarify your "What?" question, I'll try to be more specific, if you wouldn't mind sharing more. Are you familiar with how light reflects off of a sphere? Does it reflect evenly or not, and is that consistent with what you observe when you worked on your project?

I'm just stating that your seem to infer that reflectors on the moon prove that we've been to the moon.

No but its a very good indicator that the moon is a large object far away.

I'm stating that there doesn't need to be reflectors on the moon to shoot a laser at the moon, and most people don't know that.

I get that but again all it proves is that the moon is big and far away.

Are you familiar with how light reflects off of a sphere? Does it reflect evenly or not, and is that consistent with what you observe when you worked on your project?

Wasnt a part of the project.

On an object that large why wouldnt we expect to see uniformity in reflection, for the most part?

What inconsistency do you see in that regard?

On an object that large why wouldnt we expect to see uniformity in reflection, for the most part?

Oh, dear lord, no. That's actually a question that science is attempting to solve, but with more cart in front of the horse answers. "If this is so, then this must be the reason for it". Statements like that are purely philosophical assumptions and not based on empirical studies.

I'm not trying to argue the nature of reflectors, I was responding to the the question of "How did the reflectors get there?", and how it was an unnecessary and unproductive question.

For example, regardless of anyone's position of whether we landed on the moon or not, to ask a person who doesn't believe it, "How did we get the LEM onto the moon?", would be non-productive, as there is already an assumption that there can't be one on the moon, much less to answer how it got there, as proof that we landed on the moon. It's a bit circular of an argument.

Oh, dear lord, no. That's actually a question that science is attempting to solve, but with more cart in front of the horse answers. "If this is so, then this must be the reason for it". Statements like that are purely philosophical assumptions and not based on empirical studies.

I dont know... There seems to be very logical explanations.

https://physics.stackexchange.com/questions/288285/why-does-a-full-moon-seem-uniformly-bright-from-earth-shouldnt-it-be-dimmer-at

Science seems to have a handle on it, doesnt appear to be an unknown question.

Statements like that are purely philosophical assumptions and not based on empirical studies.

Again why you unevenly applying these empirical requirements?

We have been to the moon, we have images from space... You can see satellites zipping through the night sky...

I'm not trying to argue the nature of reflectors, I was responding to the the question of "How did the reflectors get there?", and how it was an unnecessary and unproductive question.

Doesnt really matter in the context of our discussion.

We know because of man made objects on the moon the approximate distance to those mirrors.

The moon is large and far away.

I dont know... There seems to be very logical explanations.

That's a good start, and googling it does help, so well done in looking into it.

I will say that you've based some of your opinion that you've shared in our exchange on things you knew nothing about until this conversation, and it's good to continue looking deeper, if you choose to. I'm not here trying to prove anything to you, and respect that you believe what you do. I came from the same place, and to question others' ability to discern things also would be an unfortunate assumption, until they prove it to be true. There's a big internet out there, and of course a lot of it is there to make us dumber, so yay for those going into it with that concern in mind, a perspective which I'm sure we both share.

Happy hunting and be nice to those seeking answers, please. Or don't, I'm not in charge of what anyone else does or thinks.

I'm not here trying to prove anything to you,

Thats pretty clear lol. Most of your argument is philosophical and not based in reality.

Once more that is a mighty high horse you are riding on. Do you normally talk down to people who challenge your views? Does it make you feel better about not being able to argue your points?

Now that we've reduced ourselves to ad hominem, it's probably best that we continue on with our day and make the best out of it.

Enjoy your life, there's a whole world out there to explore and I'm sure you've had your share of adventure already with all that's going on in the world these days.

Now that we've reduced ourselves to ad hominem, it's probably best that we continue on with our day and make the best out of it.

In my defense you were the first one to reduce yourself to that level, Im just following suit

:)

Enjoy your life

I certainly do, you as well.

there's a whole world out there to explore and I'm sure you've had your share of adventure already with all that's going on in the world these days.

Yep a whole infinite universe of wonder, not some terrarium that perfectly simulates a vast universe.

Later.

No worries, it happens. Thank you, see ya!

People and robots have traveled to the moon and brought back samples.

Do you know how many of those samples that came back have turned out to be bonafide fakes? If so, would that give you something to allow that claim to be not as solid as it appears to be?

Yes. I know exactly how many if the samples which came back from the moon are fake. ZERO.

There are, however, plenty of rocks which have been sold or presented as moon rocks which have turned out to be fake.

I'm not sure of your rationale, I am going to assume you are just under-informed. Buzz Aldrin and Neil Armstrong has been found, on record, giving fake moon rocks to foreign dignitaries under the guise that they were real.

It's like trying to use the existence of fake Rolex watches to prove that there is actually no such thing as a Rolex watch.

There have been over a 100 rocks given to other governments by NASA, all having since learned that they are fake. It is the "who" that is doing all of the giving which should be on the table for questioning, should it not?

And when the "who" is NASA, then the notion that "there is no such thing" could come back into play.

Indeed there were rocks which were given to people as gifts which may not have been real. However, you will find ONE example, from the Netherlands. Not over 100. The rock that ended up in that Dutch museum was given as a private gift and was donated to the museum after the recipient's death. Holland also has legitimate moon rocks from Apollo 17 in its national history museum.

This is a classic example of the conspiracy theory that is built from a nugget of truth and blown out if proportion through repeated exaggerations.

I believe you're not trying hard enough to entertain the thought, but are looking for rationalizations to shut down any one point.

Indeed there were rocks which were given to people as gifts which may not have been real. However, you will find ONE example, from the Netherlands.

I'm not basing my claim on one example, I just gave one example. The idea that someone will find just one example of something that could question the claim that space rocks exist is yours and yours alone.

https://en.wikipedia.org/wiki/Stolen_and_missing_moon_rocks

I shouldn't have to try! There should be evidence. You don't try to believe things. That leads you down a dead end if only believing what you want.

All your article shows is that various con men tried to sell fake moon rocks. Nowhere is is even suggested that moon rocks are fake or there were no rocks brought back by the Apollo missions.

All your article shows is that various con men tried to sell fake moon rocks. Nowhere is is even suggested that moon rocks are fake or there were no rocks brought back by the Apollo missions.

I think many reading this, including me, would disagree with your summary of "my articles", nor am I saying that I believe one thing or another.

I am, however, making the claim that moon rocks are not a reliable proof that we've been to the moon, because the evidence of it shows that the rocks, handled by agents and agencies of the government (NASA, Nixon, Astronauts), were found, in hundreds of occasions, to be untrustworthy as bonafide evidence, and, also puts into question such agencies' inability to account for them.

But that's simply not true. One rock given as a personal gift turned out to be fake. Some con men tried to sell some fake moon rocks. That's it. It had absolutely no bearing on the validity of the samples brought back from the Apollo missions.

Even though you can see more than 50% of their faces as they turn?

You can see satellites move around their respoective planet, go around the backside then appear on the opposite side. You could get a telescope and see it for yourself.

We can see planet turn, see features on their surface turn at an expected rate and move behind their planet only to reappear...

Also how do you even begin to address the orbit of mars.

What kind of an astronomical odd is it that the moon never turns or changes its side?

Not at all, many satellites are tidally locked, the moon isnt even unique in the solar system for this.

Its called tidal locking and its relatively common. Over millions of years a little energy is lost as the larger planet tugs at its moon slowing its rotation until its "locked"

Seriously many moons do this, its a well known phenomena.

This is a strange one. moons and planets are supposed to be spherical thanks to gravity, but our skyscrapers don't get squashed into the ground by gravity.

They're just painted on the dome maaaan

Another easy way to find curvature is to recreat Eratosthenes experiment with 3 or more sticks

This gif should point you in the right direction. On a globe we can mae 3 right angles intersect, not possible on a FE

http://i.imgur.com/oFD5ley.gifv

How did Eratosthenes account for refraction, or is refraction off the table?

Why would he need to account for refraction?

Because I thought that sunlight refracts at different angles, as it passes through the atmosphere.

And if so, it would seem reasonable that it needs to be accounted for, if you are working of angles of sunlights as a primary source of measurement.

Because I thought that sunlight refracts at different angles, as it passes through the atmosphere.

What kind of curve do you expect?

And how is a FE model immune to this?

Flat Earth researchers are trying to work with it to understand it, I'm not claiming that FE models are immune to this.

Science is great, but it involves testing hypothesis and not just parroting what is taught.

https://en.wikipedia.org/wiki/Bedford_Level_experiment

he first experiment at this site was conducted by Rowbotham in the summer of 1838. He waded into the river and used a telescope held eight inches (20 cm) above the water to watch a boat, with a flag on its mast three feet (0.91 m) above the water, row slowly away from him.[3] He reported that the vessel remained constantly in his view for the full six miles (9.7 km) to Welney bridge, whereas, had the water surface been curved with the accepted circumference of a spherical earth, the top of the mast should have been some 11 feet (3.4 m) below his line of sight. He published this observation using the pseudonym Parallax in 1849 and subsequently expanded it into a book, Earth Not a Globe, published in 1865.[4]

https://en.wikipedia.org/wiki/Bedford_Level_experiment

"However, in 1870, after adjusting Rowbotham's method to avoid the effects of atmospheric refraction, Alfred Russel Wallace found a curvature consistent with a spherical Earth."

Of course others didnt like the result so they kept doing the experiment until they got the answers they wanted

Or others used the actual science involved to recreate the experiment correctly smh

I've worked in science, and i've seen people tweak and repeat experiments until they get the result they want.

Rowbotham adjusted.

It happens today, holes in formulas are plugged with fudge factors that don't even exist. The mass calculation of the universe doesn't make sense? Oh then there must be some invisible mass we can't see! Hey presto "dark matter".

I agree with you're call on the Universe I think there is way too much expanse for any human to measure. Even the Big Bang is all based on 'questionable' theory, However the actual fact of our planet being spherical is not base on theory it is based on solid scientific research and when a couple of people push that to the side we get crackpot theories of the earth being flat.

This has been fun amongst all the bs political post here as of late

This has been fun amongst all the bs political post here as of late

Awful isn't it.

I believe all these things prove we have been lied to all our lives, the earth is flat

You are the only one in this thread who actually adressed the points I brought up..

I was convinced the earth is flat by watching all the info, I spread the info around now

You do understand that he addressed exactly zero of the points you brought up and simply agreed with you, right?

He at least agreed about the points I brought up instead of talking about something else.

Gravity bends light. I’m sure the further away you get from earths gravity is affects how the light is viewed.

I said further you move away from earths gravity the light bends differently. In a jet traveling fast light will react a certain way etc.. a balloon entering or exiting earths atmosphere will be different as well. I’m no scientist at all. But I assume everyone sees differently anyway.

So why is there no curvature?

There is.

One simple way of testing this is to watch a ship go over the horizon, sailing away from you.

It will disappear bottom first. Now if you increase your elevation even a little the boat comes back into view. This is unexplained on a FE. Elevation should have nothing to do with how far you can see.

Another good way of simply proving curvature is to do the calutlated fuel consumption of a plane or a boat on a curved vs flat plane.

It should very simply show if you have sailed (For example) 1000 miles over a flat distance which would eat less fuel than if you had sailed over a curved surface for 1000 miles.

One simple way of testing this is to watch a ship go over the horizon, sailing away from you. It will disappear bottom first.

You are claiming that when one is seeing ships dissapear with their bare eyes that they are going below the curve. This is not correct, if one takes binoculars and looks the ship will come back into view. If the ship was behind a horizon and you look with binoculars without rising your elavation the ship should not come back into view... but it does. This is clear proof that what you are seeing is not a ship going behind a curve.

, if one takes binoculars and looks the ship will come back into view.

This only bring objects into view that are already visible, have you ever noticed this only works for a few miles and not all the miles? Why cant I see Cuba from the Southern tip of Florida with a telescope? Why is it only small objects right on the horizon?

Why has no one tested this?

You also forgot to address why I can increase my elevation by even a few feet and bring objects back into view... On a FE this shouldnt work...

You also forgot to address why I can increase my elevation by even a few feet and bring objects back into view... On a FE this shouldnt work...

It would work. Perspective works in such a way that if for example you are looking at a long hall the floor goes up and the ceiling comes down and they meet in the middle and this line in which they meet is called horizon line. When you rise up you see more of the land (or water in your case) before it meets the sky in the horizon point.

This only bring objects into view that are already visible, have you ever noticed this only works for a few miles and not all the miles?

It brings objects into view that were invisible to the eye. You were saying earlier that when a ship goes invisible for your eye it goes beyond the curve. It doesnt.

You cant see cuba from florida because air is not clear, it distorts light. In a fog you cant see even that far.

Why has no one tested this?

People have tested on smaller scales and they see way longer than it should be possible if the earths curve is as much as is claimed.

Perspective works in such a way that if for example you are looking at a long hall the floor goes up and the ceiling comes down and they meet in the middle and this line in which they meet is called horizon line. When you rise up you see more of the land (or water in your case) before it meets the sky in the horizon point.

Lets say the boat is shaped like a cube for kicks and giggles.

The surface area of the boat doesnt change but it still will reappear in your vision if you increase your horizon. This is also replicable with a digital camera or telescope, increasing elevation increases the amount those items can see.

This is not possible on a FE.

It brings objects into view that were invisible to the eye. You were saying earlier that when a ship goes invisible for your eye it goes beyond the curve. It doesnt.

I would argue those objects are still visible just too small to see without assistance. They would be invisible totally if completely over the horizon. Otherwise on clear days we should be able to see more than a few miles away, right? But its always the same thing curvature hides objects and increasing elevation brings them back into view.

People have tested on smaller scales and they see way longer than it should be possible if the earths curve is as much as is claimed.

Then why no scientific papers? Why no quantification? Why no measurements or equations? Whats the hold up? Traditional science has mountains of research but FE only has youtbue videos?

This is not possible on a FE.

I just explained how it is possible and you didnt even adress that.

I would argue those objects are still visible just too small to see without assistance.

And I could argue that everything is visible but just not to be seen without assistence. This is useless arguing.

Then why no scientific papers? Why no quantification? Why no measurements or equations? Whats the hold up? Traditional science has mountains of research but FE only has youtbue videos?

Traditional science does not have anyone doing these measurements in real life, in actual reality. Or if you find any studies, please show.

But its always the same thing curvature hides objects and increasing elevation brings them back into view.

The vanishing point of your perspective makes things vanish before they would even go beyond the supposed curve. So what hides things is your perspective.

If you were standing on a ball, you would never see the horizon at your eye level because no matter in what direction you look, all sides of the ball are bending down and away from you. Yet even if we climb to high altitudes the horizon stays on eye level. This is not possible on a ball because the horizon would not even be on eye level when you were on the ground, and it would sink down even further the more elavated you become.

I just explained how it is possible and you didnt even adress that.

What you wrote doesnt make sense... It honestly seems like you made it up. Ive taken drawing classes and Ive never heard what you just wrote.

Can you link me to a write up on that? Any additional resource?

Traditional science does not have anyone doing these measurements in real life, in actual reality. Or if you find any studies, please show.

? I dont follow... There are mountains of evidence and peer review. Why do you not hold FE to the same level?

The vanishing point

Follow my logic here. What is the vanishing point?

I dont think you completely understand your own argument.

Yet even if we climb to high altitudes the horizon stays on eye level.

Thats not true. People have shown its not true with a theodolite. A device that tests exactly that...

I dont follow... There are mountains of evidence and peer review. Why do you not hold FE to the same level?

There isnt. There is some guy that put up some sticks ages ago up and after that people just believe in it. There is math but people havent actually done these things in real life. Like went in some place and looked how far they see from that place and then calculate the curve and see how far they should have seen and if these two things match. The people that have done this report that they see farther than they should.

Can you link me to a write up on that? Any additional resource?

Search on youtube for perspective on flat earth.

Follow my logic here. What is the vanishing point?

The Vanishing point is the horizon. Where the earth and the sky come together and meet as the horizon line.

Here is a picture: https://springsemester2015artz363.files.wordpress.com/2015/01/112-one-pt-3.jpg

Thats not true. People have shown its not true with a theodolite. A device that tests exactly that...

It should be nowhere near your eyelevel. You should be looking down. A lot. If you rise up 10 meters the horizon should go down at least ten meters. It can not go down less than ten meters because also the higher you go over a ball the farther you see and the farther you see the more the ball bends down and the lower the horizon is. If you think the amounts we are speaking of are something that need to be measured by a theodolity you are not understanding what it means to rise up on a ball.

There isnt. There is some guy that put up some sticks ages ago up and after that people just believe in it. There is math but people havent actually done these things in real life.

Youre making shit up... This is first year space science stuff, you literally do this test...

Like went in some place and looked how far they see from that place and then calculate the curve and see how far they should have seen and if these two things match. The people that have done this report that they see farther than they should.

Most of the times those tests are flawed... There is a reason no one quantifies this stuff, there is a reason there are no FE technical write ups.

Seriously, go find me the technical write up on FE. They dont exist.

Ask yourself why they dont exist.

Search on youtube for perspective on flat earth.

Lol, no like the name of this phenomena or where I can read more about it. Ive never heard of it and again Ive taken a few drawing classes in my younger days and no one ever explained this to us.

Here is a picture: https://springsemester2015artz363.files.wordpress.com/2015/01/112-one-pt-3.jpg

PERFECT! Alright so why does the vanishing point move when elevation changes? Its the same distance.

Your last paragraph never challenges my position about the theodolite... Why?

Also where are your calculations? You say it should be more, how do you know?

This is first year space science stuff, you literally do this test...

The test where you actually measure stuff in reality, not with sticks...

PERFECT! Alright so why does the vanishing point move when elevation changes?

It doesnt. No matter at what elavation you are, if you look straight ahead at the horizon, it is on your eye level. You dont need to tilt your head down.

Your last paragraph never challenges my position about the theodolite... Why?

Because the theodolite measures small changes and there should be a huge change.

Also where are your calculations? You say it should be more, how do you know?

You dont need calculations, use common sense. Imagine you are standing over a ball. The whole surface of the ball is under you. The horizon line is where the surface and the sky meet in perspective. If you rise up a meter, the WHOLE ball is now a meter below you. If you rise up 10 meters, the WHOLE ball is now under you. How can the whole ball drop 10 meters under you but the horizon line which is where the surface of the ball and the sky meet in perspective does not drop?

The test where you actually measure stuff in reality, not with sticks...

How is that not a great introduction to this theory? Start off with the basics.

You would take issue with higher level tests, right?

It doesnt. No matter at what elavation you are, if you look straight ahead at the horizon, it is on your eye level. You dont need to tilt your head down.

No its not, we went over this. Look up what a theodolite is and how you can prove how far about the horizon you are.

A simple test would be level a flat surface when you go on a hike up a tall mountain and observe how far below the horizon is. I guarantee it wont be level.

Because the theodolite measures small changes and there should be a huge change.

How large by your calculations and show your math.

You dont need calculations, use common sense. Imagine you are standing over a ball. The whole surface of the ball is under you. The horizon line is where the surface and the sky meet in perspective. If you rise up a meter, the WHOLE ball is now a meter below you. If you rise up 10 meters, the WHOLE ball is now 10 meters under you. How can the whole ball drop 10 meters under you but the horizon line which is where the surface of the ball and the sky meet in perspective does not drop?

I think you are forgetting how large this ball is...

No its not, we went over this.

I dont care about your theodolity, if I see the curvature is right in front of me on eye level and I dont need to tilt my head down, I will trust that.

A simple test would be level a flat surface when you go on a hike up a tall mountain and observe how far below the horizon is. I guarantee it wont be level.

I have no idea what you mean but ever horizon I have ever seen has been on eye level and has been level.

I think you are forgetting how large this ball is...

It does not matter how large the ball is, if you rise 100 meters above it, you are 100 meters above the whole ball. The whole damn ball will be 100 meters below you, how come the horizon which is where the ground (of the ball that was supposed to be 100 meters below you) is seen to be on eye level?

I dont care about your theodolity,

You mean you dont care about facts. Noted.

if I see the curvature is right in front of me on eye level and I dont need to tilt my head down, I will trust that.

Right... So you also dont believe in virology or bacteria? Right? Youve never seen them and you seem to only trust your sense, right?

I have no idea what you mean but ever horizon I have ever seen has been on eye level and has been level.

Test this yourself lol. Climb a tall mountain, bring a sensitive level and a piece of wood. Set the wood to be level now bring your eyes as close to level with the plane and you will note the horizon is below it.

if you rise 100 meters above it, you are 100 meters above the whole ball.

Right... So we are 100 meters up looking several miles away. What does that angle look like on paper? Write it out if you have to.

What you wrote doesnt make sense... It honestly seems like you made it up. Ive taken drawing classes and Ive never heard what you just wrote.

That's strange. Perspective is very important in architectural drawings. So you say you never heard of this "perspective" you don't even want to name in your own comment, while you boast about being an expert on the subject? phony

https://en.wikipedia.org/wiki/Perspective_%28graphical%29 https://en.wikipedia.org/wiki/Architectural_drawing#Architectural_perspective

"Perspective in drawing is an approximate representation on a flat surface of an image as it is perceived by the eye. The key concepts here are:

Perspective is the view from a particular fixed viewpoint. Horizontal and vertical edges in the object are represented by horizontals and verticals in the drawing. Lines leading away into the distance appear to converge at a vanishing point. All horizontals converge to a point on the horizon, which is a horizontal line at eye level. Verticals converge to a point either above or below the horizon."

''In art, especially painting, aerial perspective refers to the technique of creating an illusion of depth by depicting distant objects as paler, less detailed, and usually bluer than near objects.

(One caution: in common speech, the words perspective and viewpoint tend to be used interchangeably; however, in art, aerial perspective does not imply an aerial viewpoint.''

''The two most characteristic features of perspective are that objects are smaller as their distance from the observer increases; and that they are subject to foreshortening, meaning that an object's dimensions along the line of sight are shorter than its dimensions across the line of sight.''

One simple way of testing this is to watch a ship go over the horizon, sailing away from you.

Please continue looking deeper to the curvature question, it will at least allow you to let go of this notion, as we have all been "taught" this when we were younger. Heck, ask yourself where and how you've "learned" this supposition, first of all.

Let me ask you this, would knowing the distance to the horizon, and how to calculate the curvature of the Earth, be purposeful? If so, do you recommend that we use that, as that seems like a measurable claim, repeatable and non-arbitrary.

Do you use that knowledge to back your claim, or does it not require the ability to know distances and measurements?

Please continue looking deeper to the curvature question, it will at least allow you to let go of this notion, as we have all been "taught" this when we were younger. Heck, ask yourself where and how you've "learned" this supposition, first of all.

Lol Ive done a lot of research (If you want to call it that...) into FE, Im very well educated on the subject. Lets not start this off with ad hominems, eh?

Let me ask you this, would knowing the distance to the horizon, and how to calculate the curvature of the Earth, be purposeful? If so, do you recommend that we use that, as that seems like a measurable claim, repeatable and non-arbitrary.

It is measurable, but you also have to acknowledge refraction is a thing. We know it exists and we know it can affect light.

If you performed most FE "tests" in a controlled environment they would fail or give results that prove a globe earth.

There is a reason there are no FE science papers with rigorous testing, because thay all fall apart.

Lol Ive done a lot of research (If you want to call it that...) into FE, Im very well educated on the subject. Lets not start this off with ad hominems, eh?

No ad hominem in my statement was intended, I was truly offering a moment for pause, as I had to ask myself the same question once.

So, from your educated position, what formula(s) do you use to measure? This will help others gain the discernment that you have and will allow them to look at it from a scientific approach (thanks in advance).

And, I do acknowledge that refraction is a thing, we are in agreement there.

So, from your educated position, what formula(s) do you use to measure?

The formulas are on wikipedia. Really easy to look up, the math is above me though.

What measurements have you done and how do they differ from the expected result?

Im really curious to see what contradictions you have found with the current paradigm.

Thank you, I didn't ask for the formulas (and they are not on wikipedia), I was curious on what formulas you've used, so as to understand/validate what you mean when you say that you are educated on the subject, and to help you give that statement more credibility.

I was curious on what formulas you've used, so as to understand/validate what you mean when you say that you are educated on the subject, and to help you give that statement more credibility.

I Support the current paradigm, I believe the Earth is a globe.

All the info I have is currently easily available.

Im curious as to why you dont bevel it. What tests have you run and how were the results different that what science would have anticipated.

Where do you find the discrepancy with the current system?

What led you to believe there is anything wrong with current theories?

I appreciate the question.

I was told all of my life to accept scientific expertise on all matters, especially of the cosmos. I was told this by being ridiculed for asking innocent questions, out of my own ignorance. It turns out that I was aware enough to see deception in the answers, at least.

As you are here on a conspiracy sub, you also surely question things. At some point, I'm sure we would agree that there is deception happening at institutional levels, whether it be media institutions, scientific, educational, financial, political.

The punchline is, whatever "conspiracies" you have followed and "believe" in, whatever scientific facts you've learned through your life, I also have, and likely share a lot of that knowledge with you. I've just stopped accepting a lot of the contradictions, assumptions, and rationalizations and started looking into it myself.

I could spend hours with you sharing my story, and would love to, if you were truly interested. There is no one discrepancy in the current system, there is a multitude. The notion that we can use curvature to defend a globe earth is one of them, we can't. The only way to prove it is to show it from space. There are a lot of discrepancies for what we are being showed from space.

The current "system" in which there are discrepancies, isn't just referring to orbital mechanics, astrophysics, and earth science, it refers to the system of control that is inferred by almost every post on this subreddit.

When people on this sub ask, "What's the conspiracy?", the answer is usually one and the same: OP infers that there are unspoken systems of control that deceive the public (which I tend to accept as true). Try it on for yourself, look at any r/conspiracy post, and see if that answer could be inferred.

Nevertheless, regarding your question to me, the information is out there, yes there is plenty of bad information, but an intelligent mind, as yours surely is, will be able to find good info, if you are truly curious. You surely do that for other conspiracies, I'm assuming, as, you are investing time on a conspiracy sub.

There is no one discrepancy in the current system, there is a multitude.

Lets just stick with gravity since thats the topic at hand.

What is one discrepancy with gravity that you have noticed?

it refers to the system of control that is inferred by almost every post on this subreddit.

We could have that debate but Im more interested in these discrepancies. Just toss a few out.

The question about censorship and control on websites like this is independent of hard facts about our world.

It would be like saying there is a system in place that forces people to believe 2+2=5, you can say that but at the end of the say we can prove that 2+2=4 regardless of what an authority says.

It would be like saying there is a system in place that forces people to believe 2+2=5, you can say that but at the end of the say we can prove that 2+2=4 regardless of what an authority says.

I agree with you and that is what Flat Earth researchers strive to do, as every argument about the globe and space came from an authority claiming 2+2=5, in their opinion, which, again, must acknowledge the larger system at play, as, if the larger system can control what the masses think and know about everything, history included, then we could all have been born into a world of deception and not have known it.

My concern with gravity starts with learning about gravity and not just believing it. Newtonian gravity, which is what we all celebrate, was disproven and replaced by Einstein's theory of gravity. Many would hold the opinion that Einstein didn't disprove it, he just improved it. It is an acknowledgement that everything up to the point of Einstein, regarding gravity, didn't work as postulated by Newton.

So, the argument then is that for hundreds of years, a view of gravity that was not functional was good enough to celebrate and say that newton proved gravity. It reminds me of a joke regarding someone who claimed that they were as smart as DaVinci, because he also created plans for a helicopter that did not and could not fly.

"Proofs" of gravity seems to put the cart before the horse. For example, before the supposed discovery of gravitational waves this year, at one time in the past, they didn't have an answer to 96% of what should be out there IF gravity existed. So they had to name that 96% dark matter and dark energy. They did not "discover" these elements of nature, they had to name a thing couldn't explain the failure of gravity's "math".

as every argument about the globe and space came from an authority claiming 2+2=5,

False. You can prove a globe Earth without NASA or the government. Im not sure what led you to believe all data comes from NASA.

"Proofs" of gravity seems to put the cart before the horse. For example, before the supposed discovery of gravitational waves this year, at one time in the past, they didn't have an answer to 96% of what should be out there IF gravity existed. So they had to name that 96% dark matter and dark energy. They did not "discover" these elements of nature, they had to name a thing couldn't explain the failure of gravity's "math".

Im sorry but this doesnt actually seem to address the argument at hand. If anything you just praised the scientific process for doing what its supposed to do.

As for proofs putting the cart before the horse, Ill have to disagree.

We know exaclty how objects interact with each other under gravity. Can you offer an alternative that can model our universe even remotely close to what we observe?

I offered you a lot of things for us to discuss that I haven't really heard you back your claim to, other than a position and set of opinions that anyone could invoke with any previous knowledge on any of the subject matter.

My confidence in your ability to speak beyond what is learned by watching Bill Nye the Science Guy is diminishing, to be honest. So I'll leave you with this:

There is plenty out there about the world to learn about, if you're not interested in it, it's no harm to anyone to let them investigate. It is integral to the knowledge of the larger systems of corruption, so the idea that we must chastise those who seek and ask questions, or that it is a PSYop to distract and to make the "truth" community look bad, is an unfortunate position to take.

For those out there reading any of this, they can decide for themselves whether or not fallacious tactics were employed one way or another and can discern for themselves what they choose to give their energy to.

My confidence in your ability to speak beyond what is learned by watching Bill Nye the Science Guy is diminishing, to be honest. So I'll leave you with this:

Once more argue the person if you cant argue the topic

:)

There is plenty out there about the world to learn about, if you're not interested in it, it's no harm to anyone to let them investigate.

Investigate away and Ill be right there pointing out the flaws.

so the idea that we must chastise those who seek and ask questions, or that it is a PSYop to distract and to make the "truth" community look bad, is an unfortunate position to take.

If it means anything to you I also think that the FE movement is bad for humanity as a whole. It reinforces anti-intellectualism and teaches people to abandon the scientific method i n lieu of their feels. You literally have to ignore entire sciences and arguments to make a FE work. I could go on and on about the issues with FE but since you would rather sling mud and make up bunk positions with no support Im out.

Keep reading and I dont mean just watching youtube videos (Its pretty clear that is where a bulk of your "knowledge" comes from) but challenge you own viewpoints dont blindly accept this bullshit.

For those out there reading any of this, they can decide for themselves whether or not fallacious tactics were employed one way or another and can discern for themselves what they choose to give their energy to.

I agree with that sentiment but I also think you should step down off that soap box a little and worry more about how much you dont understand about your own universe

:)

Peace out.

Peace out. It's great that those that have come before us and those that will come after can reflect on our exchange, so I'm grateful for your participation. They can read your position and read mine and decide for themselves, we both know that neither one of us was looking to convince each other.

Maybe it’s because our eyes shape.. so we get to the extent of our peripheral and it puts it together as curved but a camera doesn’t. Idk...

Maybe the further you move away from gravity the light isn’t bent, or for that lens.. no clouds.. or reflective light. I have zero explanation as I am not scientist.. just saying I never just assume something I don’t understand is correct or incorrect.

Some would call that a straw man argument, unless you are telling me that you are unaware that there are a multitude of high altitude balloon shots from non-rectilinear lenses (or footage with the videos corrected for the fish-eye effect, unless you'd also tell me that you don't believe in those).

You should probably look into this if you hadn't already. If you have, please feel free to explain footage from a non-rectilinear lens.

Dude don't waste your time with these shills, there is NO CURVATURE.

I don't remember the name of the story, but there is an old Edgar Allen Poe short story about a man who pilots hot air balloons and he goes into extreme detail about the measurements and science involved and even describes a phenomenon where at a certain height the surface of the earth appears to be concave.

I'm sure it would be a valuable read to someone of your beliefs.

I am not sure what the value is in reading a fictional writer's story of a man that allegedly had something to say a long time ago.

Wouldn't it be more valuable to seek observations from any hot air balloon pilots that are still alive today?

Don't read it then.

Fair enough.

I have said this many times.. they can show me articles of whatever. I don’t believe any of it.. I just don’t trust anything and don’t believe shit. Just try to make logical reasoning in my own head. I don’t have a stand on either side here.. just don’t put my money on anything being real..

But just on the topic of that specific lens camera.. I could easily say they didn’t use that type of lens.. as the other side of the argument they did.. where is proof.. I’ve never seen these cameras to be honest.. I didn’t do the experiment I’ve already stated I don’t put my money anywhere.. I don’t believe shit.

I dont believe anything either. I think this existence is holographic. It certainly is not what I thought it was and I am not who I thought I was.

I was someone before that was funneled into a way of life and thinking. Basically who my parents raised me to be.. forced into church etc.. but as you get older you start to wonder what’s going on. I’m also investigative and like to problem solve and when things don’t add up or make sense and people want me to believe things it’s super tough. When secrets are held from the people I also just believe that there is nothing we know as real.

Ultimately we can know only what we are.

I think our minds show us what’s going on.. whether we choose to listen or not.. that’s the question.

One often hears flat earthers say that there is no curve.

No curve on the horizon, but the ground is curved toward the horizon.

That if you calculate the curve and go test it out in real life, you can see things in the horizon that you should not be able to see if there was a curve.

Seems like some real easy science to cite. Links?

And that bodies of water dont curve either and that some laser measurements over large canals show this.

Bodies of water, minus wind and gravity waves, are level with the center of gravity of Earth. Earth is an oblate spheroid, so the top of large bodies of water will be tangent to the center of gravity. That means curved.

And that bodies of water dont curve either and that some laser measurements over large canals show this.

Again, seems like some easy science to point to. Links? Large bodies of water curve with the tangent to Earth's center of gravity. This is easily demonstrated by standing on the beach in Key West, looking south, and NOT seeing Cuba, just 90 miles away. Meanwhile, you can stand 100 miles from Mt. Rainier in Washington and see its top. So there is no "perspective" or "atmospheric" limitation. The limitation is that water is in the way because you have to look down to see Cuba.

And that one can see distances on sea that one should not be able to see if there was a curve.

More uncited claims.

Also the balloons that have been sent up with cameras that dont have fish eye lenses show a horizon that does not curve.

The horizon will not be curved. The horizon is a 2-d shape. A circle. You are at the center of that circle. It will not curve, it will be flat. The ground, however, will be curving away from you to the horizon.

Again. And this cannot be stressed enough. THE HORIZON IS FLAT. IT IS A CIRCLE. It isn't going to bend away from you, it's going to be flat, because it's a circle and you are at the center of the circle.

What do you think?

I think I'm wasting my time.

How is this sitting at zero with 246 comments yet the rest of the sub is full of upvoted politics. Sad.

So why is there no curvature?

Science doesnt explain the lack of curvature it explains THE curvature and might even go to the extent of explaining how a lazer is able to appear to be going in a "straight" line...i dunno just sayin'

How do you explain a flat earth but every other plannet and moon in the universe is a sphere?

Is the sun inside our atmosphere?

The ability to build machines to go to the moon still exists. But the old technology which was originally used has been destroyed/lost. The factories were re-purposed. The moulds for the parts were destroyed. The machines built today would use different materials and different components so we simply wouldn't be able to build a Saturn V today. Well, you could, but it would be prohibitively expensive and a bit of a waste of time.

In this case (with you denying historical and scientific fact), I have the luxury of being both on the side of truth and being right.

There is no way in which the moon module can ascend to the surface of the moon while not leaving any marks in the dust like surface of the moon.

And you can't prove your own existence either can you? Gravity is a force which can be observed thanks to the way it affects objects which interact with it. You can't directly observe magnetism but I don't hear you saying magnets aren't real (although you could be one of the ICP if your understanding of science is as it seems).

Another easy way to find curvature is to recreat Eratosthenes experiment with 3 or more sticks

This gif should point you in the right direction. On a globe we can mae 3 right angles intersect, not possible on a FE

http://i.imgur.com/oFD5ley.gifv

What kind of an astronomical odd is it that the moon never turns or changes its side?

I dont follow... There are mountains of evidence and peer review. Why do you not hold FE to the same level?

There isnt. There is some guy that put up some sticks ages ago up and after that people just believe in it. There is math but people havent actually done these things in real life. Like went in some place and looked how far they see from that place and then calculate the curve and see how far they should have seen and if these two things match. The people that have done this report that they see farther than they should.

Can you link me to a write up on that? Any additional resource?

Search on youtube for perspective on flat earth.

Follow my logic here. What is the vanishing point?

The Vanishing point is the horizon. Where the earth and the sky come together and meet as the horizon line.

Here is a picture: https://springsemester2015artz363.files.wordpress.com/2015/01/112-one-pt-3.jpg

Thats not true. People have shown its not true with a theodolite. A device that tests exactly that...

It should be nowhere near your eyelevel. You should be looking down. A lot. If you rise up 10 meters the horizon should go down at least ten meters. It can not go down less than ten meters because also the higher you go over a ball the farther you see and the farther you see the more the ball bends down and the lower the horizon is. If you think the amounts we are speaking of are something that need to be measured by a theodolity you are not understanding what it means to rise up on a ball.

Of course you have! You're claiming that light acts in a way which goes against our current understanding. All that science we've conducted in space is fake, so there's actually now no evidence for all that physics we've been using.

You're claiming that light acts in a way which goes against our current understanding.

Its still photons.

All that science we've conducted in space is fake, so there's actually now no evidence for all that physics we've been using.

Physics wasnt invented in space.

Theories have been wrong.

unless you think there's a better way of studying the universe than observation and constant testing and experimenting?

If anything I would like more actual observation and experimentation.

No one has observed gravity, we cant isolate it as a force and observe it. Its totally invisible to us.

So how do you account for light which clearly comes from a long way away?

Just because its of a different frequency does not imply distance.

How do you account for countless observations of the universe?

Like lights in the sky?

You throw in something like a flat Earth and the non-existence of gravity and all of our observations are null and void. All that science collapses.

Only the space science.

The point was to talk about the points that were brought up in the original post.

Oh, dear lord, no. That's actually a question that science is attempting to solve, but with more cart in front of the horse answers. "If this is so, then this must be the reason for it". Statements like that are purely philosophical assumptions and not based on empirical studies.

I dont know... There seems to be very logical explanations.

https://physics.stackexchange.com/questions/288285/why-does-a-full-moon-seem-uniformly-bright-from-earth-shouldnt-it-be-dimmer-at

Science seems to have a handle on it, doesnt appear to be an unknown question.

Statements like that are purely philosophical assumptions and not based on empirical studies.

Again why you unevenly applying these empirical requirements?

We have been to the moon, we have images from space... You can see satellites zipping through the night sky...

I'm not trying to argue the nature of reflectors, I was responding to the the question of "How did the reflectors get there?", and how it was an unnecessary and unproductive question.

Doesnt really matter in the context of our discussion.

We know because of man made objects on the moon the approximate distance to those mirrors.

The moon is large and far away.

No, but you can't use a fatuous argument and not expect to have it thrown back at you. There's no proof of flat Earth. You haven't provided anything. You say you can't observe the curvature of the Earth, but only because you choose to ignore or deny the evidence. You've no proof of anything else. No pictures of the edge. No explanation for the observable universe. Nothing.

Now that we've reduced ourselves to ad hominem, it's probably best that we continue on with our day and make the best out of it.

Enjoy your life, there's a whole world out there to explore and I'm sure you've had your share of adventure already with all that's going on in the world these days.

What you wrote doesnt make sense... It honestly seems like you made it up. Ive taken drawing classes and Ive never heard what you just wrote.

That's strange. Perspective is very important in architectural drawings. So you say you never heard of this "perspective" you don't even want to name in your own comment, while you boast about being an expert on the subject? phony

https://en.wikipedia.org/wiki/Perspective_%28graphical%29 https://en.wikipedia.org/wiki/Architectural_drawing#Architectural_perspective

"Perspective in drawing is an approximate representation on a flat surface of an image as it is perceived by the eye. The key concepts here are:

Perspective is the view from a particular fixed viewpoint. Horizontal and vertical edges in the object are represented by horizontals and verticals in the drawing. Lines leading away into the distance appear to converge at a vanishing point. All horizontals converge to a point on the horizon, which is a horizontal line at eye level. Verticals converge to a point either above or below the horizon."

''In art, especially painting, aerial perspective refers to the technique of creating an illusion of depth by depicting distant objects as paler, less detailed, and usually bluer than near objects.

(One caution: in common speech, the words perspective and viewpoint tend to be used interchangeably; however, in art, aerial perspective does not imply an aerial viewpoint.''

''The two most characteristic features of perspective are that objects are smaller as their distance from the observer increases; and that they are subject to foreshortening, meaning that an object's dimensions along the line of sight are shorter than its dimensions across the line of sight.''

Isn't it strange how objects on earth don't attract each other? Take a large heavy object (lots of gravity in that) and put some small items next to it, they don't fly to it and stick.

Even in space the moon stays in an orbit, it doesn't get closer and closer until it collides with the earth, which it should since they are both attracting each other.

The "keeps missing the ground" explanation of orbiting is amusing, gravitational force acts on something but it doesn't get any faster or closer.

I've worked in science, and i've seen people tweak and repeat experiments until they get the result they want.

Rowbotham adjusted.

It happens today, holes in formulas are plugged with fudge factors that don't even exist. The mass calculation of the universe doesn't make sense? Oh then there must be some invisible mass we can't see! Hey presto "dark matter".

That's good info, thanks. I've had enough Echo 1 for today, thanks for the shares. And thanks for your other contributions ITT, I was reading up on some of them.

Also, here's something I've stumbled upon, trying to look into passively bouncing signals, until I decided I was over it, for the night.

I'm not sure if there's anything good in it, but maybe put it in your back pocket for a rainy day to see if it offers up anything about anything, whether or not it speaks to the topic at hand.

http://www.dtic.mil/dtic/tr/fulltext/u2/675512.pdf

I just realized that, when I was 16, I read a book from the hitchhiker's guide series (or one of Douglas Adam's books, at least), and distinctly recall that it was mentioned, in a metaphysically humorous way, that the secret to flying is to jump off of something and then miss the ground.

I think it's time to revisit that series, I was just talking about it at a halloween party the other night, coincidentally.